a cr from Gmat

This topic has expert replies
Legendary Member
Posts: 1119
Joined: Fri May 07, 2010 8:50 am
Thanked: 29 times
Followed by:3 members

a cr from Gmat

by diebeatsthegmat » Thu Sep 01, 2011 12:39 am
Noting the success of DVDs in the marketplace, the newly opened video store Movie Tapes, which sells specialty videocassettes, began an ambitious publicity campaign to promote the sale of their tapes over more technologically advanced DVDs. After six months in business, Movie Tapes found that its sales had doubled - in fact, it had sold twice as many copies of movies on tape as all the other stores in town had sold on DVD of the same movies. Clearly, the publicity campaign was responsible for Movie Tapes' video cassettes selling better than DVDs.

Which of the following, if true, casts the most serious doubt on the conclusion drawn above?

DVDs include special features that enhance the viewing experience.
The local school system uses only VCRs in the classroom.
Only a very small number of the movies the store sells are available on DVD.
A month before the campaign began, the public library started buying videocassettes to lend.
Neither the price of DVDs nor that of videocassettes has risen in two years.

i said D for the answer and they posted C as the answer and why??? please help

Legendary Member
Posts: 581
Joined: Sun Apr 03, 2011 7:53 am
Thanked: 52 times
Followed by:5 members

by killer1387 » Thu Sep 01, 2011 2:07 am
its a bouncer. IMO D.

what is the source of the question??

Legendary Member
Posts: 1119
Joined: Fri May 07, 2010 8:50 am
Thanked: 29 times
Followed by:3 members

by diebeatsthegmat » Thu Sep 01, 2011 11:41 am
killer1387 wrote:its a bouncer. IMO D.

what is the source of the question??
posted.. its from the forum of gmatclub

User avatar
GMAT Instructor
Posts: 2193
Joined: Mon Feb 22, 2010 6:30 pm
Location: Vermont and Boston, MA
Thanked: 1186 times
Followed by:512 members
GMAT Score:770

by David@VeritasPrep » Thu Sep 01, 2011 12:57 pm
This question above seems to be based on the following LSAT question. The question below is a strengthening where the one above is weakening, but you can see that it same argument structure. I will explain this question after some discussion.


December 1995 LSAT Logical Reasoning Section 2, Question 12.


One year ago a local government initiated an antismoking advertising campaign in local newspapers, which it financed by imposing a tax on cigarettes of 20 cents per pack. One year later, the number of people in the locality who smoke cigarettes had declined by 3 percent. Clearly, what was said in the advertisements had an effect, although a small one, on the number of people in the locality who smoke cigarettes.

Which one of the following, if true, most helps to
strengthen the argument?

(A) Residents of the locality have not increased their use of other tobacco products such as snuff and chewing tobacco since the campaign went into effect.

(B) A substantial number of cigarette smokers in the locality who did not quit smoking during the campaign now smoke less than they did before it began.

(C) Admissions to the local hospital for chronic respiratory ailments were down by 15 percent one year after the campaign began.

(D) Merchants in the locality responded to the local tax by reducing the price at which they sold cigarettes by 20 cents per pack.

(E) Smokers in the locality had incomes that on average were 25 percent lower than those of nonsmokers.
Veritas Prep | GMAT Instructor

Veritas Prep Reviews
Save $100 off any live Veritas Prep GMAT Course

User avatar
Master | Next Rank: 500 Posts
Posts: 398
Joined: Tue Jul 26, 2011 11:39 pm
Location: India
Thanked: 41 times
Followed by:6 members

by prateek_guy2004 » Thu Sep 01, 2011 2:07 pm
I can see the question stem where are the options.........?
Don't look for the incorrect things that you have done rather look for remedies....

https://www.beatthegmat.com/motivation-t90253.html

User avatar
GMAT Instructor
Posts: 2193
Joined: Mon Feb 22, 2010 6:30 pm
Location: Vermont and Boston, MA
Thanked: 1186 times
Followed by:512 members
GMAT Score:770

by David@VeritasPrep » Thu Sep 01, 2011 2:50 pm
Don't focus on the question at the top of the posting. It is very similar to the LSAT question that I just posted to this thread. Look at that one and you will better understand what the earlier question is doing.
Veritas Prep | GMAT Instructor

Veritas Prep Reviews
Save $100 off any live Veritas Prep GMAT Course

Master | Next Rank: 500 Posts
Posts: 135
Joined: Tue Oct 13, 2009 10:27 am
Thanked: 3 times

by boazkhan » Thu Sep 01, 2011 9:54 pm
for the 1st question IMO: C

for the 2nd question IMO: D

Legendary Member
Posts: 1119
Joined: Fri May 07, 2010 8:50 am
Thanked: 29 times
Followed by:3 members

by diebeatsthegmat » Fri Sep 02, 2011 12:26 am
David@VeritasPrep wrote:This question above seems to be based on the following LSAT question. The question below is a strengthening where the one above is weakening, but you can see that it same argument structure. I will explain this question after some discussion.


December 1995 LSAT Logical Reasoning Section 2, Question 12.


One year ago a local government initiated an antismoking advertising campaign in local newspapers, which it financed by imposing a tax on cigarettes of 20 cents per pack. One year later, the number of people in the locality who smoke cigarettes had declined by 3 percent. Clearly, what was said in the advertisements had an effect, although a small one, on the number of people in the locality who smoke cigarettes.

Which one of the following, if true, most helps to
strengthen the argument?

(A) Residents of the locality have not increased their use of other tobacco products such as snuff and chewing tobacco since the campaign went into effect.

(B) A substantial number of cigarette smokers in the locality who did not quit smoking during the campaign now smoke less than they did before it began.

(C) Admissions to the local hospital for chronic respiratory ailments were down by 15 percent one year after the campaign began.

(D) Merchants in the locality responded to the local tax by reducing the price at which they sold cigarettes by 20 cents per pack.

(E) Smokers in the locality had incomes that on average were 25 percent lower than those of nonsmokers.
argument in the cr is that the local government want to decrease the number of smokers by an antismoking advertising campaign: imposing a tax on cigarettles of 20 cent per pact and one year later, as the result, the number of people who smoke had decreased 3 percent
conlusion: the campaign worked,

question: strenthen the conclusion that the local government campaign worked .
B is my choice...
A could be but i chose B

Senior | Next Rank: 100 Posts
Posts: 82
Joined: Sat Mar 26, 2011 4:59 am
Thanked: 4 times

by B166418 » Fri Sep 02, 2011 12:31 am
David could you please assist us with OA for both the questions

In 1st question it has been given that sale of videos cassettes has been doubled,this point is not justified by OPTION C

Legendary Member
Posts: 608
Joined: Sun Jun 19, 2011 11:16 am
Thanked: 37 times
Followed by:8 members

by saketk » Fri Sep 02, 2011 9:24 am
Hi David , I chose B for the question you posted.
Kindly let me know what's the answer. Also, could you please explain the question posted earlier in this thread. I chose C for that one.

Newbie | Next Rank: 10 Posts
Posts: 2
Joined: Sun Aug 28, 2011 12:41 pm

by yogi234567890 » Fri Sep 02, 2011 11:13 am
i dont think C would weaken the argument as "it had sold twice as many copies of movies on tape as all the other stores in town had sold on DVD of the same movies"
clearly says the comparision is btw same movies on dvd and video cass.

User avatar
GMAT Instructor
Posts: 2193
Joined: Mon Feb 22, 2010 6:30 pm
Location: Vermont and Boston, MA
Thanked: 1186 times
Followed by:512 members
GMAT Score:770

by David@VeritasPrep » Fri Sep 02, 2011 12:23 pm
For the LSAT question that I posted above the official answer is D.

This the type of question where you are looking for an alternate cause - or in the case of the LSAT question I posted you are looking to block an alternate cause and thereby to strengthen the cause named in the argument.

For the LSAT question, the conclusion is that the advertisements affected the behavior of the smokers (some quit smoking). So we are saying the ads were the cause of the drop in smoking. Now the LSAT question asks us to strengthen the ads as the cause. This can be done by blocking an alternate cause. The possible alternate cause is the 20 cent tax. Choice D blocks the tax as a cause of people quitting smoking because the tax was not passed on to the people.

Finding an alternate cause is a classic way to weaken cause and effect and blocking an alternate cause is a classic way to strengthen cause and effect.

For the LSAT question, B would not be a correct answer choice because of the simple fact that you cannot explain and effect with another effect. The argument tells us that some people quit smoking. Choice B tells us that others who did not quit smoke less. This is not an alternate cause, but rather is another effect that still needs to be explained. Now we need to explain what caused some people to quit smoking and others to smoke less. We still do not know if it was the ads that did this.

Choice A is a good option since it blocks on of the possible alternate explanations for the people who quit smoking. Maybe those people switched to other tobacco products. This is nearly as good as D. However, D is implicated by the argument since it talks about the 20 cent tax. This makes it a more likely alternate cause than A is.
Veritas Prep | GMAT Instructor

Veritas Prep Reviews
Save $100 off any live Veritas Prep GMAT Course

User avatar
GMAT Instructor
Posts: 2193
Joined: Mon Feb 22, 2010 6:30 pm
Location: Vermont and Boston, MA
Thanked: 1186 times
Followed by:512 members
GMAT Score:770

by David@VeritasPrep » Fri Sep 02, 2011 12:35 pm
For the earlier posted question - D is the best answer.

(DieBeatstheGMAT - perhaps the answer was posted wrong when it said C?)

Do you see how the original question that starting this posting is very much the same as the LSAT question I posted??

The earlier question is a weakening question while the LSAT is a strengthening but the structure of the argument is the same.

We are told that Movie Tapes has started an campaign. (Sounds very much like the LSAT question with the anti-smoking ads). We are told that the number of tapes sold had doubled. The conclusion then is that the publicity campaign was the cause.

Do you see why I brought in the LSAT question? Basically the same cause and effect (although the LSAT question is a bit better edited.)

So now we can weaken with an alternate cause for the the doubling of the sales.

The choices are not labeled but from their positions:

A) Goes the opposite direction - would tend to explain why more DVDs are sold not more of the tapes.

B) But does the local school system buy tapes? Are they buying more tapes?

C) Seems intriguing - but are these tapes popular? Maybe that is why they are not on DVD. It requires an extra assumption to pick this choice namely that people want the tapes that are not on DVD - so this is not the best option. Also, the stimulus says they sold twice as many copies on tape as other stores sold on DVD "Of the same movies" so we are not even talking about movies that are not available on DVD.

D) Not perfect but the best answer. This is an alternate cause. If the library is buying lots of cassettes that can be the explanation for increased sales.

E) This does not provide a difference to explain why tapes are selling more than DVDs.

So D is certainly the best choice .
Veritas Prep | GMAT Instructor

Veritas Prep Reviews
Save $100 off any live Veritas Prep GMAT Course

Legendary Member
Posts: 608
Joined: Sun Jun 19, 2011 11:16 am
Thanked: 37 times
Followed by:8 members

by saketk » Fri Sep 02, 2011 1:09 pm
For the LSAT question, B would not be a correct answer choice because of the simple fact that you cannot explain and effect with another effect.
This is a very good learning for me... Thanks

David -- Can you please let me know whether we should attempt LSAT questions for GMAT? I agree that those questions are very intriguing and are beneficial, but do we really need all that for GMAT?

I read Ron's post somewhere in this forum where he advised to first master the GMAT questions.

Kindly share your thoughts.

User avatar
Master | Next Rank: 500 Posts
Posts: 398
Joined: Tue Jul 26, 2011 11:39 pm
Location: India
Thanked: 41 times
Followed by:6 members

by prateek_guy2004 » Fri Sep 02, 2011 2:22 pm
Ohk at first glance i thought its E reason because they question stem says that smokers have quit smoking because of to hike in the cost...

and statement E says

Smokers in the locality had incomes that on average were 25 percent lower than those of nonsmokers

So because of the low income they left smoking..now this is going no where and has introduced a new reason behind it...

But yeah as you said the best way for a strengthen or weakening ques is to block or choose a alternate cause....So clearly D is the new and alternate cause....

Thanks David it was a nice ques..
Don't look for the incorrect things that you have done rather look for remedies....

https://www.beatthegmat.com/motivation-t90253.html